You are on page 1of 1

Luc TARTAR to John MACKEY, October 28, 2009

Your Putnam set 10 (of October 27).


Your problem 1: Let a = 77 , b = 7a , and c = 7b . For showing that c − b is a multiple of 120, one proves that
it is a multiple of 8, of 3, and of 5. Modulo 8, 7 = −1 (mod 8), and every odd power of 7, like a, b, and c
is −1 (mod 8). Modulo 3, 7 = 1 (mod 3), and every power of 7, like a, b, and c is 1 (mod 3).
Since 74 = 1 (mod 5), one is led to work modulo 4 and modulo 5, i.e. modulo 20: one has 72 = 49 = 9
(mod 20), 73 = 63 = 3 (mod 20), and 74 = 21 = 1 (mod 20), so that for any nonnegative integer n, one
has 720n+3 = 73 = 3 (mod 20), giving a = b = c = 3 (mod 20), and c − b is a multiple of 5.
Your problemP2: One has 1 ∈ S, since a1 > 0. Let k be the smallest element of S in {2, . . . , n}: if it does not
P value ∈ {0, . . . , n−k} such that ak +. . .+ak+r > 0,
exist one has j∈S aj > 0; if k exists and r is the smallest
the claim is that X = {k, . . . , k + r} ⊂ S, so that j∈X aj > 0: it is obvious if r = 0, and if r ≥ 1 and
0 < s ≤ r, then one has ak + . . . + ak+s−1 ≤ 0 by definition of r being the smallest, and this implies that
ak+s + . . . + ak+r > 0, so that k + s ∈ S. Then one repeats the analysis with the smallest element of S in
{k + r + 1, . . . , n}, and in that way one decomposes S into disjoint intervals such that the sum of the aj for
j in one of theses intervals is > 0, proving the desired result.
Your problem 3: If f (x + 1) = f (x) + 1 and g(x + 1) = g(x) + 1 for all x ∈ R, then h = f ◦ g has the
same property, so that f (n) has the property. Then f (n) (x) − x has period 1, and since it is continuous it
attains its minimum an . Because f (m+n) (x) = f (m) f (n) (x) , if one writes y = f (n) (x) one has f (m+n) (x) −
x = f (m) (y) − y + f (n) (x) − x, and since f (m) (y) − y ≥ am and f (n) (x) − x ≥ an , one deduces that
f (m+n) (x) − x ≥ am + an , and after taking the minimum in x one deduces that am+n ≥ am + an for all
positive integers m, n.
Any sequence satisfying am+n ≥ am + an for all positive integers m, n is such that the limit of ann exists.
By induction, am+k n ≥ am + k an for all k ≥ 1. For any N one has N = m + k n for some m ∈ {0, . . . , n − 1}
by Euclidean division, so that aNN ≥ am+k m +k an
n , and when N → ∞ one has k → ∞, and m+k n → n
am +k an an

since there are only a finite number of values of m and for each choice the sequence am+k m +k an
n tends to n .
an
aN an
One deduces that lim inf N →∞ N ≥ n for all n ≥ 1, and one lets now n tend to ∞, and one obtains
lim inf N →∞ aNN ≥ lim supN →∞ aNN , so that the limit exists, and it may be +∞, as for the case an = n2 for
all n, but this does not happen for the sequence created with f : one defines bn = maxx (f (n) (x) − x), and
one shows that bm+n ≤ bm + bn , and then an ≤ bn ≤ n b1 so that limn→∞ ann ≤ b1 .
R∞ 2
Your problem 4: By the change of variable x = y 2 the integral becomes I = 2 0 y4y+1 dy. Since y 4 +1 = (y 2 +
√ √ √ √ R∞
1)2 − 2y 2 = (y 2 − 2y + 1) (y 2 + 2y + 1), and y2 −√12y+1 − y2 +√12y+1 = 2y4 +1 2y
, one has 2I = 0 y2 −√y2y+1 −
√ √ 2

√y dy. The quantity y22y− √ 2 − 2y+ √ 2 has integral 0, since it is the derivative of log yy2 −√2y+1 , and

y 2 +√2y+1 − 2y+1 y 2 + 2y+1 + 2y+1
y 2 −√2y+1
√ R∞ √ √
√2 √2

2
y + 2y+1
tends to 1 as y tends to 0 or to ∞, and one deduces that 2 2I = 0 2
y − 2y+1
+ 2
y + 2y+1
dy =
√ R +∞ 1 z−1 2
√ 1
2 1 1+z 2
2 −∞ y2 +√2y+1 dy. By the change of variable y = √2 , one has y + 2y + 1 = y + √2 + 2 = 2 ,
√ R +∞
and one then has 2 2I = 2 −∞ z2dz+1 = 2π, so that I = √π2 .
Your problem 5: One writes 1 = 12 + 12 , 12 = 13 + 16 , 16 = 17 + 42 1
, and so on, based on k1 = k+1 1 1
+ k (k+1) ,
1 1
so that one constructs solutions of x1 + . . . + xn = 1 with 1 ≤ x1 ≤ . . . ≤ xn , and the highest values are
successively, 1, 2, 6, 42, etc., while the en values are 2, 3, 7, 43, etc., always one above, so that the proposed
sequence en cannot be improved. Actually, the preceding construction gives e11 + . . . + e1n = 1 − e1 e21···en ,
which one proves by induction: it is true for n = 1, and if it is true for n one writes e1 e2 · · · en = en+1 − 1
and en+11 −1 = en+1 1 1
+ (en+1 −1) en+1 , and since (en+1 − 1) en+1 = e1 · · · en+1 , it is true for n + 1.
If for 2 ≤ x1 ≤ . . . ≤ xn+1 , one has x11 + . . . + xn+1 1
= 1 but xn+1 ≥ en+1 , i.e. xn+1 > e1 · · · en (or
1 1 1 1 1 1 1 1
xn+1 < e1 ···en ), one deduces that x1 +. . .+ xn > e1 +. . .+ en , so that there exists i ∈ {1, . . . , n} with xi > ei ,
i.e. xi < ei . One chooses i to be the smallest possible, which cannot be i = 1 since one cannot have x1 = 1,
and then one has x1 = e1 , . . . , xi−1 = ei−1 , and xi < ei , so that 1 > x11 + . . . + x1i = e11 + . . . + ei−1
1
+ x1i =
1 1 1 1 1 1
1 − e1 ···ei−1 + xi , but since xi ≤ ei − 1 = e1 · · · ei−1 , one has xi ≥ e1 ···ei−1 , giving 1 − e1 ···ei−1 + xi ≥ 1, a
contradiction.

You might also like